Pagina 1 di 1

Massimi e minimi senza derivate

Inviato: 25 giu 2011, 10:11
da bĕlcōlŏn
Come al solito, nel questionario della seconda prova di Matematica per il Licei Scientifici, compaiono alcuni problemi di massimo e minimo.
Quesito 1 (Scientifico Tradizionale): Trovare fra tutti i cilindri inscritti in una sfera di raggio $ 60 $ cm quello di volume massimo.
Quesito 2 (Scientifico PNI/Tradizione): Si trovi il punti della curva $ y=\sqrt{x} $ più vicino a quello di coordinate $ (4;0) $.
Quesito 6 (Scientifico PNI): Di tutti i coni inscritti in una sfera di raggio $ 10 $ cm, qual è quello di superficie laterale massima?

Risolveteli senza usare le derivate :)

Re: Massimi e minimi senza derivate

Inviato: 25 giu 2011, 12:16
da Mike
in effetti, il quesito due l'ho risolto nel modo seguente (non sono maturando) :

cercare il punto di quella curva più vicino equivale a cercare la più piccola circonferenza di centro (4,0), che abbia punti in comune con la nostra curva.
consideriamo pertanto un fascio di circonferenze di centro (4,0) e mettiamolo a sistema con la nostra buona curva.
$ y= \sqrt{x} $
$ x^2 +y^2 -8x + 16 = r^2 $

che, sostituendo la prima nella seconda, ci porta a

$ x^2 - 7x + 16 - r^2 = 0 $

A questo punto imponiamo che la circonferenza sia tangente (dimodoché abbia un solo punto in comune), eguagliando il delta a 0:

$ 49 - 64 + 4r^2 = 0 $
$ 4r^2 = 15 $
$ r^2 = \frac{15}{4} $

ora sostituiamo $ r^2 $ nell'equazione e troviamo la soluzione, che è il punto $ (\frac{7}{2}, \sqrt{\frac{7}{2}}) $

Re: Massimi e minimi senza derivate

Inviato: 25 giu 2011, 22:28
da Drago96
Sto cercando di fare il primo con AM-GM ma non viene! :(
Sono abbastanza sicuro di $V=\pi h(60+h/2)(60-h/2)$ e dunque per massimizzare devo avere ${h+120\over 3}=\sqrt[3]{h(60+h/2)(60-h/2)}$
Ma non riesco a risolvere l'ultima equazione... (e nemmeno wolfram alpha, il che significa che devo aver sbagliato qualcosa, ma non capisco cosa...)

Re: Massimi e minimi senza derivate

Inviato: 26 giu 2011, 01:10
da xXStephXx
Io ho chiamato $ r $ il raggio di base del cilindro.
Quindi ho che $ \sqrt{r^{2}+\frac{h^{2}}{4}}=60 $ e $ \frac{r^{2}h}{3}\pi $ deve essere massimo.

Quindi $ \frac{r^{2}+\frac{h^{2}}{4}}{2} \geq \sqrt{\frac{r^{2}h^{2}}{4}} $
Dopo un po' di conti arrivo a:

$ 4320000\pi \geq \frac{r^{2}h^{2}}{3}\pi $
Il problema è che per trovare il volume dovrei dividere tutto per $ h $, quindi non ottengo un numero preciso.

[Edit] Ho provato ora a fare un sistema a due incognite cercando sia il raggio sia l'altezza mettendo a sistema le due considerazioni fatte, ma siccome mi vengono numeri decimali (calcolatrice), forse è sbagliata anche la considerazione fatta con AM-GM.

OT: ma come mai il codice $ \LaTeX $ mi esce così piccolo?

Re: Massimi e minimi senza derivate

Inviato: 26 giu 2011, 10:42
da Drago96
Non capisco da dove arriva il "diviso 3" nella formula del volume... non è un cilindro?
Inoltre In AM-GM come mai metti un $h^2$ se devi massimizzare $r^2h$ ?? :?

Comunque qualcuno mi dice dove sbaglio? Perchè ormai dopo un bel po' di tentativi mi sono convinto di quello che ho scritto...

P.S: per gli esponenti fatti da un solo carattere non servono le graffe... ;)

Re: Massimi e minimi senza derivate

Inviato: 26 giu 2011, 10:46
da xXStephXx
Hai ragione il fratto $ 3 $ non ci sta. (perchè l'ho calcolata come se fosse un cono?) xD
Ho messo $ h^2 $ perchè non sapevo come toglierlo. Ora riprovo.

Re: Massimi e minimi senza derivate

Inviato: 26 giu 2011, 11:11
da Veluca
Drago96 ha scritto:Comunque qualcuno mi dice dove sbaglio? Perchè ormai dopo un bel po' di tentativi mi sono convinto di quello che ho scritto...
Il problema è che, come hai fatto am-gm tu, non trovi il volume massimo perchè lasci una "h" nell'AM, prova a farla sparire :)
Testo nascosto:
$\displaystyle V=\frac{\pi}{\sqrt[3]3}\sqrt[3]{h\cdot3\left(60-\frac h2\right)\cdot\left(60+\frac h2\right)}=\frac{\pi}{\sqrt[3]3}\frac{h+180-\frac32h+60+\frac h2}3=\frac{\pi}{\sqrt[3]3}80=\pi\frac{80\sqrt[3]9}3$
Drago96 ha scritto:P.S: per gli esponenti fatti da un solo carattere non servono le graffe... ;)
Ma è buona abitudine metterle per non dimenticarle quando servono :D

Re: Massimi e minimi senza derivate

Inviato: 26 giu 2011, 13:37
da exodd
@ Veluca: Anche così, non rispetti le condizioni di uguaglianza! :wink:

$ V=\pi\frac{2}{(1+\sqrt{3})(2+\sqrt{3})}[(\frac{1+\sqrt{3}}{2}h)(2+\sqrt{3})(60-\frac{h}{2})(60+\frac{h}{2})]=\pi\frac{2}{(1+\sqrt{3})(2+\sqrt{3})}(\frac{60(3+\sqrt{3})}{3})^3=96000\sqrt{3}\pi $

Re: Massimi e minimi senza derivate

Inviato: 26 giu 2011, 19:41
da kalu
quesito 1
$ \displaystyle \sqrt[3]{ (\frac{r}{\sqrt{2}})( \frac{r}{\sqrt{2}})( \frac{h}{2})} \le \sqrt{\frac{({\frac{r}{\sqrt{2}}})^2+({\frac{r}{\sqrt{2}} })^2+(\frac{h}{2})^2 }{3}} $; ma $ \displaystyle r^2+ \frac{h^2}{4}=R^2 $, quindi $ \displaystyle V \le \frac{4 \pi R^3 }{\sqrt{27}} $. Si ha l'uguaglianza quando $ h=sqrt{2}r $.

quesito 2
soluzione identica a quella di Mike (ma sospetto che belcolon ne abbia una migliore :wink: )

quesito 6
Applicando il secondo teorema di Euclide (come?) si ottiene che $ r=\sqrt{h(2R-h)} $, quindi $ S_l=\pi h \sqrt{2R(2R-h)} $.
$ \displaystyle \sqrt[4]{ (\frac{h}{2}) (\frac{h}{2}) (\frac{2R}{3}) (2R-h) } \le \frac{(\frac{h}{2}) +(\frac{h}{2})+(\frac{2R}{3})+(2R-h) }{4} $, da cui $ \displaystyle S_l \le \frac{8\pi R^2}{\sqrt{27}} $. Si ha l'uguaglianza quando $ \displaystyle h=\frac{4R}{3} $.

Re: Massimi e minimi senza derivate

Inviato: 26 giu 2011, 20:02
da xXStephXx
Non sono stato io a fare il quesito 2 :D

Re: Massimi e minimi senza derivate

Inviato: 26 giu 2011, 20:23
da kalu
Mmm... al quesito 6 ho commesso degli errori, proverò a rimediare. OK, finalmente così dovrebbe andare (avevo scambiato un raggio con un diametro :oops: ). Scusa Mike per averti tolto il merito di aver risolto il 2 :mrgreen: Edito.

Re: Massimi e minimi senza derivate

Inviato: 27 giu 2011, 17:47
da paga92aren
Detto $h$ l'altezza del cono, $r$ il raggio di base, $l$ il lato e $R$ il raggio della sfera, si ha che:
1) $l^2=2Rh$
2) $r^2=l^2-h^2$
e sapendo che la superficie laterale è $S=\pi lr$ ottengo $S=\pi \sqrt{2Rh^2(2R-h)}$ e massimizzo $h^2(2R-h)=4\frac{h}{2}\frac{h}{2}(2R-h)\leq 4\left( \frac{\frac{h}{2}+\frac{h}{2}+(2R-h)}{3}\right)^3=\frac{32}{27}R^3$ con point of incident $h=\frac{4R}{3}$. Sapendo $S\leq \pi \sqrt{\frac{64}{27}R^4}=\frac{8}{9}\sqrt{3}R^2\pi$

Re: Massimi e minimi senza derivate

Inviato: 12 lug 2011, 20:40
da bĕlcōlŏn
kalu ha scritto:quesito 1
$ \displaystyle \sqrt[3]{\left(\frac{r}{\sqrt{2}}\right)\left(\frac{r}{\sqrt{2}}\right)\left(\frac{h}{2}\right)} \le \sqrt{\frac{\left({\frac{r}{\sqrt{2}}}\right)^2+\left({\frac{r}{\sqrt{2}} }\right)^2+\left(\frac{h}{2}\right)^2 }{3}} $; ma $ \displaystyle r^2+ \frac{h^2}{4}=R^2 $, quindi $ \displaystyle V \le \frac{4 \pi R^3 }{\sqrt{27}} $. Si ha l'uguaglianza quando $ h=\sqrt{2}r $.
Mike ha scritto: Cercare il punto di quella curva più vicino equivale a cercare la più piccola circonferenza di centro (4,0), che abbia punti in comune con la nostra curva.
consideriamo pertanto un fascio di circonferenze di centro (4,0) e mettiamolo a sistema con la nostra buona curva.
$ y= \sqrt{x} $
$ x^2 +y^2 -8x + 16 = r^2 $
che, sostituendo la prima nella seconda, ci porta a
$ x^2 - 7x + 16 - r^2 = 0 $
A questo punto imponiamo che la circonferenza sia tangente (dimodoché abbia un solo punto in comune), eguagliando il delta a 0:
$ 49 - 64 + 4r^2 = 0 $
$ 4r^2 = 15 $
$ r^2 = \frac{15}{4} $
ora sostituiamo $ r^2 $ nell'equazione e troviamo la soluzione, che è il punto $ (\frac{7}{2}, \sqrt{\frac{7}{2}}) $
paga92aren ha scritto:Detto $h$ l'altezza del cono, $r$ il raggio di base, $l$ il lato e $R$ il raggio della sfera, si ha che:
1) $l^2=2Rh$
2) $r^2=l^2-h^2$
e sapendo che la superficie laterale è $S=\pi lr$ ottengo $S=\pi \sqrt{2Rh^2(2R-h)}$ e massimizzo $h^2(2R-h)=4\frac{h}{2}\frac{h}{2}(2R-h)\leq 4\left( \frac{\frac{h}{2}+\frac{h}{2}+(2R-h)}{3}\right)^3=\frac{32}{27}R^3$ con point of incident $h=\frac{4R}{3}$. Sapendo $S\leq \pi \sqrt{\frac{64}{27}R^4}=\frac{8}{9}\sqrt{3}R^2\pi$
Per avere sott'occhio tutte e tre le soluzioni. In effetti per il secondo esercizio, prendo un generico punto $A(x,\sqrt{x})$ sulla curva con $x\geq 0$. La distanza da $B(4,0)$ è $AB=\sqrt{(x-4)^2+x}=\sqrt{x^2-7x+16}$. Ora basta trovare il "punto più basso della parabola" con $x\geq 0$. L'ascissa del vertice è $\dfrac{7}{2} \geq 0$ e quindi è questo il minimo cercato.

Per il metodo usato nel terzo c'è una simpatica generalizzazione. Dati $n$ numeri reali positivi la cui somma è fissata, il prodotto $\displaystyle\prod_{i=1}^n x_i^{a_i}$, dove gli $a_i$ sono razionali positivi è massimo quando $\dfrac{x_i}{a_i}$ è costante per ogni $i$ (in altre parole sarebbe AM-GM pesata). In quel caso, infatti, il massimo si ha quando $\dfrac{h}{2} = 2R-h \Rightarrow h=\dfrac{4}{3} R$.